If $alpha$ and $beta$ are disjoint, so are $alpha$ and $beta^k$?Show $pialphapi^{-1}$ and $pibetapi^{-1}$ are...

How to distinguish easily different soldier of ww2?

Has a sovereign Communist government ever run, and conceded loss, on a fair election?

Should I file my taxes? No income, unemployed, but paid 2k in student loan interest

What is better: yes / no radio, or simple checkbox?

How would an energy-based "projectile" blow up a spaceship?

Why do we call complex numbers “numbers” but we don’t consider 2-vectors numbers?

What exactly is the meaning of "fine wine"?

I am the light that shines in the dark

Short story about an infectious indestructible metal bar?

What can I do if someone tampers with my SSH public key?

Will the concrete slab in a partially heated shed conduct a lot of heat to the unconditioned area?

Is it a Cyclops number? "Nobody" knows!

How does learning spells work when leveling a multiclass character?

Are small insurances worth it?

Inorganic chemistry handbook with reaction lists

Giving a talk in my old university, how prominently should I tell students my salary?

Sort array by month and year

Ultrafilters as a double dual

What is Tony Stark injecting into himself in Iron Man 3?

Can Witch Sight see through Mirror Image?

Was it really inappropriate to write a pull request for the company I interviewed with?

Interpretation of linear regression interaction term plot

Use Mercury as quenching liquid for swords?

Did Amazon pay $0 in taxes last year?



If $alpha$ and $beta$ are disjoint, so are $alpha$ and $beta^k$?


Show $pialphapi^{-1}$ and $pibetapi^{-1}$ are disjoint given $alpha$ and $beta$ are disjoint for an arbitrary permutation $pi$Cycles of odd length: $alpha^2=beta^2 implies alpha=beta$Order of a $alpha beta^{n/q}$ given the order of $alpha$ and $beta$If $alpha,betain S_n$ are disjoint, are $alpha^m, beta^n$ disjoint for any $m,ninBbb{N}$?Conditions on $alpha, beta$ under which $A rtimes_{alpha} B $ and $A rtimes_{beta} B$ are isomorphic$(alpha,beta)^{-1}=beta^{-1}alpha^{-1}$, proofdisjoint permutations commutesHow to prove that $sign(alpha beta) = sign(alpha)sign(beta)$?If $beta$ is the transposition $beta = (1, 4)$, compute both $betaalpha$ and $betaalphabeta^{−1}$ and compute orders.Commuting permutations are multiples of each other













1












$begingroup$



If $alpha$ and $beta$ are disjoint permutations, so are $alpha$ and $beta^k$.




I'm working on the group of permutations of a finite set $X$ with composition. The power is defined as usual using composition.



I don't know if it's true, I just found this while proving another thing. Disjoint meaning that if $alpha(x)=x$ then $beta(x)ne x$ and vice versa.



I'm trying by induction. The base case is direct. Assuming that $beta$ and $alpha^{k-1}$ are disjoint, let $iin X$. If $beta(i)ne i$ then $alpha^{k-1}(i)=i$ and $alpha(i)=i$, so aplying $alpha$ in the first equality we get $alpha^k(i) = i$.



But I'm a bit stuck in the case $beta(i)=i$. We would have $alpha^{k-1}(i)ne i$ and $alpha(i) ne i$. Assuming that $alpha^k(i) = i$ was the case then $alpha^{k-1}(alpha(i)) = i$ then $beta(alpha(i)) = alpha(i)$ but this seems to give me nothing.



Any help will be appreciated.










share|cite|improve this question









$endgroup$

















    1












    $begingroup$



    If $alpha$ and $beta$ are disjoint permutations, so are $alpha$ and $beta^k$.




    I'm working on the group of permutations of a finite set $X$ with composition. The power is defined as usual using composition.



    I don't know if it's true, I just found this while proving another thing. Disjoint meaning that if $alpha(x)=x$ then $beta(x)ne x$ and vice versa.



    I'm trying by induction. The base case is direct. Assuming that $beta$ and $alpha^{k-1}$ are disjoint, let $iin X$. If $beta(i)ne i$ then $alpha^{k-1}(i)=i$ and $alpha(i)=i$, so aplying $alpha$ in the first equality we get $alpha^k(i) = i$.



    But I'm a bit stuck in the case $beta(i)=i$. We would have $alpha^{k-1}(i)ne i$ and $alpha(i) ne i$. Assuming that $alpha^k(i) = i$ was the case then $alpha^{k-1}(alpha(i)) = i$ then $beta(alpha(i)) = alpha(i)$ but this seems to give me nothing.



    Any help will be appreciated.










    share|cite|improve this question









    $endgroup$















      1












      1








      1





      $begingroup$



      If $alpha$ and $beta$ are disjoint permutations, so are $alpha$ and $beta^k$.




      I'm working on the group of permutations of a finite set $X$ with composition. The power is defined as usual using composition.



      I don't know if it's true, I just found this while proving another thing. Disjoint meaning that if $alpha(x)=x$ then $beta(x)ne x$ and vice versa.



      I'm trying by induction. The base case is direct. Assuming that $beta$ and $alpha^{k-1}$ are disjoint, let $iin X$. If $beta(i)ne i$ then $alpha^{k-1}(i)=i$ and $alpha(i)=i$, so aplying $alpha$ in the first equality we get $alpha^k(i) = i$.



      But I'm a bit stuck in the case $beta(i)=i$. We would have $alpha^{k-1}(i)ne i$ and $alpha(i) ne i$. Assuming that $alpha^k(i) = i$ was the case then $alpha^{k-1}(alpha(i)) = i$ then $beta(alpha(i)) = alpha(i)$ but this seems to give me nothing.



      Any help will be appreciated.










      share|cite|improve this question









      $endgroup$





      If $alpha$ and $beta$ are disjoint permutations, so are $alpha$ and $beta^k$.




      I'm working on the group of permutations of a finite set $X$ with composition. The power is defined as usual using composition.



      I don't know if it's true, I just found this while proving another thing. Disjoint meaning that if $alpha(x)=x$ then $beta(x)ne x$ and vice versa.



      I'm trying by induction. The base case is direct. Assuming that $beta$ and $alpha^{k-1}$ are disjoint, let $iin X$. If $beta(i)ne i$ then $alpha^{k-1}(i)=i$ and $alpha(i)=i$, so aplying $alpha$ in the first equality we get $alpha^k(i) = i$.



      But I'm a bit stuck in the case $beta(i)=i$. We would have $alpha^{k-1}(i)ne i$ and $alpha(i) ne i$. Assuming that $alpha^k(i) = i$ was the case then $alpha^{k-1}(alpha(i)) = i$ then $beta(alpha(i)) = alpha(i)$ but this seems to give me nothing.



      Any help will be appreciated.







      group-theory symmetric-groups






      share|cite|improve this question













      share|cite|improve this question











      share|cite|improve this question




      share|cite|improve this question










      asked yesterday









      AnalyticHarmonyAnalyticHarmony

      657313




      657313






















          2 Answers
          2






          active

          oldest

          votes


















          1












          $begingroup$

          The statement to be proved is untrue under your definition of disjoint:




          Your Defintion (Disjoint): $alpha$ and $beta$ are disjoint if $alpha(x)=x$ implies $beta(x)neq x$ and vice versa.




          To see this, consider $alpha=(1;2)(3)(4)$ and $beta=(1)(2)(3;4)$, $k=2$.



          You get a correct result is you instead use




          Defintion (Disjoint): $alpha$ and $beta$ are disjoint if $alpha(x)neq x$ implies $beta(x)= x$ and vice versa.




          To see this, suppose $alpha(x)neq x$. Then disjointness implies $beta(x)=x$, so $beta^k(x)=beta^{k-1}(x)=dots=beta(x)=x$. On the other hand, if $beta^k(x)neq x$, then it must be the case that $beta(x)neq x$ (since $beta(x)=ximplies beta^k(x)=x$ as shown before), so disjointness implies $alpha(x)=x$. This proves $alpha$ and $beta^k$ are disjoint.






          share|cite|improve this answer









          $endgroup$













          • $begingroup$
            Downvoter, care to explain?
            $endgroup$
            – Mike Earnest
            yesterday



















          3












          $begingroup$

          The set of elements that is moved by $beta$ (let's call it $B$) is disjoint with the set of elements that is moved by $alpha$ (let's call it $A$) so if you apply $beta$ total of $k$ times, you still only move things in $B$, that is disjoint from $A$.






          share|cite|improve this answer









          $endgroup$













            Your Answer





            StackExchange.ifUsing("editor", function () {
            return StackExchange.using("mathjaxEditing", function () {
            StackExchange.MarkdownEditor.creationCallbacks.add(function (editor, postfix) {
            StackExchange.mathjaxEditing.prepareWmdForMathJax(editor, postfix, [["$", "$"], ["\\(","\\)"]]);
            });
            });
            }, "mathjax-editing");

            StackExchange.ready(function() {
            var channelOptions = {
            tags: "".split(" "),
            id: "69"
            };
            initTagRenderer("".split(" "), "".split(" "), channelOptions);

            StackExchange.using("externalEditor", function() {
            // Have to fire editor after snippets, if snippets enabled
            if (StackExchange.settings.snippets.snippetsEnabled) {
            StackExchange.using("snippets", function() {
            createEditor();
            });
            }
            else {
            createEditor();
            }
            });

            function createEditor() {
            StackExchange.prepareEditor({
            heartbeatType: 'answer',
            autoActivateHeartbeat: false,
            convertImagesToLinks: true,
            noModals: true,
            showLowRepImageUploadWarning: true,
            reputationToPostImages: 10,
            bindNavPrevention: true,
            postfix: "",
            imageUploader: {
            brandingHtml: "Powered by u003ca class="icon-imgur-white" href="https://imgur.com/"u003eu003c/au003e",
            contentPolicyHtml: "User contributions licensed under u003ca href="https://creativecommons.org/licenses/by-sa/3.0/"u003ecc by-sa 3.0 with attribution requiredu003c/au003e u003ca href="https://stackoverflow.com/legal/content-policy"u003e(content policy)u003c/au003e",
            allowUrls: true
            },
            noCode: true, onDemand: true,
            discardSelector: ".discard-answer"
            ,immediatelyShowMarkdownHelp:true
            });


            }
            });














            draft saved

            draft discarded


















            StackExchange.ready(
            function () {
            StackExchange.openid.initPostLogin('.new-post-login', 'https%3a%2f%2fmath.stackexchange.com%2fquestions%2f3139292%2fif-alpha-and-beta-are-disjoint-so-are-alpha-and-betak%23new-answer', 'question_page');
            }
            );

            Post as a guest















            Required, but never shown

























            2 Answers
            2






            active

            oldest

            votes








            2 Answers
            2






            active

            oldest

            votes









            active

            oldest

            votes






            active

            oldest

            votes









            1












            $begingroup$

            The statement to be proved is untrue under your definition of disjoint:




            Your Defintion (Disjoint): $alpha$ and $beta$ are disjoint if $alpha(x)=x$ implies $beta(x)neq x$ and vice versa.




            To see this, consider $alpha=(1;2)(3)(4)$ and $beta=(1)(2)(3;4)$, $k=2$.



            You get a correct result is you instead use




            Defintion (Disjoint): $alpha$ and $beta$ are disjoint if $alpha(x)neq x$ implies $beta(x)= x$ and vice versa.




            To see this, suppose $alpha(x)neq x$. Then disjointness implies $beta(x)=x$, so $beta^k(x)=beta^{k-1}(x)=dots=beta(x)=x$. On the other hand, if $beta^k(x)neq x$, then it must be the case that $beta(x)neq x$ (since $beta(x)=ximplies beta^k(x)=x$ as shown before), so disjointness implies $alpha(x)=x$. This proves $alpha$ and $beta^k$ are disjoint.






            share|cite|improve this answer









            $endgroup$













            • $begingroup$
              Downvoter, care to explain?
              $endgroup$
              – Mike Earnest
              yesterday
















            1












            $begingroup$

            The statement to be proved is untrue under your definition of disjoint:




            Your Defintion (Disjoint): $alpha$ and $beta$ are disjoint if $alpha(x)=x$ implies $beta(x)neq x$ and vice versa.




            To see this, consider $alpha=(1;2)(3)(4)$ and $beta=(1)(2)(3;4)$, $k=2$.



            You get a correct result is you instead use




            Defintion (Disjoint): $alpha$ and $beta$ are disjoint if $alpha(x)neq x$ implies $beta(x)= x$ and vice versa.




            To see this, suppose $alpha(x)neq x$. Then disjointness implies $beta(x)=x$, so $beta^k(x)=beta^{k-1}(x)=dots=beta(x)=x$. On the other hand, if $beta^k(x)neq x$, then it must be the case that $beta(x)neq x$ (since $beta(x)=ximplies beta^k(x)=x$ as shown before), so disjointness implies $alpha(x)=x$. This proves $alpha$ and $beta^k$ are disjoint.






            share|cite|improve this answer









            $endgroup$













            • $begingroup$
              Downvoter, care to explain?
              $endgroup$
              – Mike Earnest
              yesterday














            1












            1








            1





            $begingroup$

            The statement to be proved is untrue under your definition of disjoint:




            Your Defintion (Disjoint): $alpha$ and $beta$ are disjoint if $alpha(x)=x$ implies $beta(x)neq x$ and vice versa.




            To see this, consider $alpha=(1;2)(3)(4)$ and $beta=(1)(2)(3;4)$, $k=2$.



            You get a correct result is you instead use




            Defintion (Disjoint): $alpha$ and $beta$ are disjoint if $alpha(x)neq x$ implies $beta(x)= x$ and vice versa.




            To see this, suppose $alpha(x)neq x$. Then disjointness implies $beta(x)=x$, so $beta^k(x)=beta^{k-1}(x)=dots=beta(x)=x$. On the other hand, if $beta^k(x)neq x$, then it must be the case that $beta(x)neq x$ (since $beta(x)=ximplies beta^k(x)=x$ as shown before), so disjointness implies $alpha(x)=x$. This proves $alpha$ and $beta^k$ are disjoint.






            share|cite|improve this answer









            $endgroup$



            The statement to be proved is untrue under your definition of disjoint:




            Your Defintion (Disjoint): $alpha$ and $beta$ are disjoint if $alpha(x)=x$ implies $beta(x)neq x$ and vice versa.




            To see this, consider $alpha=(1;2)(3)(4)$ and $beta=(1)(2)(3;4)$, $k=2$.



            You get a correct result is you instead use




            Defintion (Disjoint): $alpha$ and $beta$ are disjoint if $alpha(x)neq x$ implies $beta(x)= x$ and vice versa.




            To see this, suppose $alpha(x)neq x$. Then disjointness implies $beta(x)=x$, so $beta^k(x)=beta^{k-1}(x)=dots=beta(x)=x$. On the other hand, if $beta^k(x)neq x$, then it must be the case that $beta(x)neq x$ (since $beta(x)=ximplies beta^k(x)=x$ as shown before), so disjointness implies $alpha(x)=x$. This proves $alpha$ and $beta^k$ are disjoint.







            share|cite|improve this answer












            share|cite|improve this answer



            share|cite|improve this answer










            answered yesterday









            Mike EarnestMike Earnest

            24.3k22151




            24.3k22151












            • $begingroup$
              Downvoter, care to explain?
              $endgroup$
              – Mike Earnest
              yesterday


















            • $begingroup$
              Downvoter, care to explain?
              $endgroup$
              – Mike Earnest
              yesterday
















            $begingroup$
            Downvoter, care to explain?
            $endgroup$
            – Mike Earnest
            yesterday




            $begingroup$
            Downvoter, care to explain?
            $endgroup$
            – Mike Earnest
            yesterday











            3












            $begingroup$

            The set of elements that is moved by $beta$ (let's call it $B$) is disjoint with the set of elements that is moved by $alpha$ (let's call it $A$) so if you apply $beta$ total of $k$ times, you still only move things in $B$, that is disjoint from $A$.






            share|cite|improve this answer









            $endgroup$


















              3












              $begingroup$

              The set of elements that is moved by $beta$ (let's call it $B$) is disjoint with the set of elements that is moved by $alpha$ (let's call it $A$) so if you apply $beta$ total of $k$ times, you still only move things in $B$, that is disjoint from $A$.






              share|cite|improve this answer









              $endgroup$
















                3












                3








                3





                $begingroup$

                The set of elements that is moved by $beta$ (let's call it $B$) is disjoint with the set of elements that is moved by $alpha$ (let's call it $A$) so if you apply $beta$ total of $k$ times, you still only move things in $B$, that is disjoint from $A$.






                share|cite|improve this answer









                $endgroup$



                The set of elements that is moved by $beta$ (let's call it $B$) is disjoint with the set of elements that is moved by $alpha$ (let's call it $A$) so if you apply $beta$ total of $k$ times, you still only move things in $B$, that is disjoint from $A$.







                share|cite|improve this answer












                share|cite|improve this answer



                share|cite|improve this answer










                answered yesterday









                gammagamma

                3,39221342




                3,39221342






























                    draft saved

                    draft discarded




















































                    Thanks for contributing an answer to Mathematics Stack Exchange!


                    • Please be sure to answer the question. Provide details and share your research!

                    But avoid



                    • Asking for help, clarification, or responding to other answers.

                    • Making statements based on opinion; back them up with references or personal experience.


                    Use MathJax to format equations. MathJax reference.


                    To learn more, see our tips on writing great answers.




                    draft saved


                    draft discarded














                    StackExchange.ready(
                    function () {
                    StackExchange.openid.initPostLogin('.new-post-login', 'https%3a%2f%2fmath.stackexchange.com%2fquestions%2f3139292%2fif-alpha-and-beta-are-disjoint-so-are-alpha-and-betak%23new-answer', 'question_page');
                    }
                    );

                    Post as a guest















                    Required, but never shown





















































                    Required, but never shown














                    Required, but never shown












                    Required, but never shown







                    Required, but never shown

































                    Required, but never shown














                    Required, but never shown












                    Required, but never shown







                    Required, but never shown







                    Popular posts from this blog

                    Magento 2 - Add success message with knockout Planned maintenance scheduled April 23, 2019 at 23:30 UTC (7:30pm US/Eastern) Announcing the arrival of Valued Associate #679: Cesar Manara Unicorn Meta Zoo #1: Why another podcast?Success / Error message on ajax request$.widget is not a function when loading a homepage after add custom jQuery on custom themeHow can bind jQuery to current document in Magento 2 When template load by ajaxRedirect page using plugin in Magento 2Magento 2 - Update quantity and totals of cart page without page reload?Magento 2: Quote data not loaded on knockout checkoutMagento 2 : I need to change add to cart success message after adding product into cart through pluginMagento 2.2.5 How to add additional products to cart from new checkout step?Magento 2 Add error/success message with knockoutCan't validate Post Code on checkout page

                    Fil:Tokke komm.svg

                    Where did Arya get these scars? Unicorn Meta Zoo #1: Why another podcast? Announcing the arrival of Valued Associate #679: Cesar Manara Favourite questions and answers from the 1st quarter of 2019Why did Arya refuse to end it?Has the pronunciation of Arya Stark's name changed?Has Arya forgiven people?Why did Arya Stark lose her vision?Why can Arya still use the faces?Has the Narrow Sea become narrower?Does Arya Stark know how to make poisons outside of the House of Black and White?Why did Nymeria leave Arya?Why did Arya not kill the Lannister soldiers she encountered in the Riverlands?What is the current canonical age of Sansa, Bran and Arya Stark?